edited by
1,129 views
13 votes
13 votes

Consider a matrix $A$ of dimension $m \times n$ such that -

$A x=\left[\begin{array}{l}1 \\ 1 \\ 1\end{array}\right]$ has no solutions and $A x=\left[\begin{array}{l}0 \\ 1 \\ 0\end{array}\right]$ has exactly one solution

Which of the following CAN be true?

  1. $\operatorname{Rank}(A)=2$
  2. $m=3$
  3. $n=1$
  4. $\operatorname{Rank}(A)=1$
edited by

3 Answers

13 votes
13 votes

option ; a , b, c, d are correct one . 

0 votes
0 votes

Since the question is Which of them CAN be true, it means that we have to just find one example to show that the option is possible.
 

OptionNo SolutionUnique Solution
A & B$$\begin{bmatrix} a & b & 1 \\ 0 & c & 1 \\ 0 & 0 & 1 \end{bmatrix}$$$$\begin{bmatrix} a & b & 0 \\ 0 & c & 1 \\ 0 & 0 & 0 \end{bmatrix}$$
C & D$$\begin{bmatrix} a & 1 \\ 0 & 1 \\ 0 & 1 \end{bmatrix}$$interchanging R1 and R2

$$\begin{bmatrix} a & 1 \\ 0 & 0 \\ 0 & 0 \end{bmatrix}$$
 


,where all the alphabets are non-zero numbers.


0 votes
0 votes
Since b is a 3x1 matrix obtained from Ax, we can easily infer that the dimensions of A and x must be 3xn and nx1. So, if A is a mxn matrix, where m must be equal to 3.

If we have a unique solution for b and, at the same time, no solution for another b, it means that A has linearly independent column vectors, but they are not covering the entire space. So, the number of linearly independent vectors in matrix A must be less than the dimension of b, which is less than 3. So, n should be either 2 or 1. So, the rank can also be either 2 or 1.
Answer:

Related questions

1.9k
views
3 answers
29 votes
GO Classes asked Mar 29, 2023
1,884 views
Let $A$ be an $m \times n$-matrix. Consider the system of linear equations $A X=b$, which of the following statement is always true:Suppose $m>n$ then rank ... unique solution.If $A X=b$ has a unique solution then $A$ has to be invertible.
1.4k
views
3 answers
16 votes
GO Classes asked Mar 29, 2023
1,362 views
Given a matrix $A\left(m \times n\right)$ and $A x=b$. Consider below statements :$\text{S}: m<n$\mathrm{P}:\; A$ has $ ... \text{S} \rightarrow \sim \text{P}$(S$ and $\text{P}) \rightarrow \text{R}$
867
views
2 answers
13 votes
GO Classes asked Mar 29, 2023
867 views
Choose the correct statements.If $b>\frac{2}{3}$, then $A=\left[\begin{array}{ll}2 & b \\ 3 & 1\end{array}\right]$ is invertible.If $P$ is an ... equations with $m$ equations and $n$ variables, such that $m=n,$ always has a solution.
1.4k
views
3 answers
16 votes
GO Classes asked Mar 29, 2023
1,408 views
Which of the following is/are TRUE?If the echelon form of an $m \times n$ matrix has a pivot in every column then $n \geq m$If the echelon form ... equations in $13$ unknowns has at least one solution then it has infinitely many solutions.